6
$\begingroup$

I found this problem interesting to be here:

If $a,b>0$ then find the following limit: $\lim_{x\to\pm\infty}\left(\frac{a^{\frac{1}{x}}+b^{\frac{1}{x}}}{2}\right)^x$

Thanks!

  • 0
    A [related question](http://math.stackexchange.com/questions/130497/provide-different-proofs-for-the-following-equality/).2012-12-23

1 Answers 1

6

One can use L'Hospital's Rule mechanically. The logarithm of our expression is $\frac{\log\left(\frac{a^{1/x}+b^{1/x}}{2}\right)}{1/x}.$ Differentiate top and bottom. We get $\frac{1}{\frac{a^{1/x}+b^{1/x}}{2}}(-1/x^2)\frac{\frac{a^{1/x}\log a+b^{1/x}\log b}{2}}{(-1/x^2)}$ Cancel the $(-1/x^2)$. The limit is $\dfrac{\log a+\log b}{2}$. Then exponentiate.